LSAT and Law School Admissions Forum

Get expert LSAT preparation and law school admissions advice from PowerScore Test Preparation.

User avatar
 Dave Killoran
PowerScore Staff
  • PowerScore Staff
  • Posts: 5850
  • Joined: Mar 25, 2011
|
#88549
Complete Question Explanation
(The complete setup for this game can be found here: lsat/viewtopic.php?f=182&p=88548#p88548)

The correct answer choice is (E).

Answer choice (A): This answer choice could occur under Templates #1 and #2.

Answer choice (B): This answer choice could occur under Template #1.

Answer choice (C): This answer choice could occur under Templates #1 and #2.

Answer choice (D): This answer choice could occur under Templates #1 and #2.

Answer choice (E): This is the correct answer choice. In Template #1, the latest M’s delivery could occur is third; in Template #2, M’s delivery must be last. Thus, M’s delivery can never be fourth.
 bethavedon
  • Posts: 7
  • Joined: Jun 20, 2016
|
#28579
Hi,

I'm struggling to figure out how to begin attacking this one. My set-up has two templates: Temp. 1: with M before H & K and Temp. 2 : with H & K before M

That led me to have Temp 1 as: M - H - G - K (and F before G, but can't really draw that in here)

and Temp 2 as: L - H - G - K - M (and also F - G and a line showing H - M)

Not sure if I'm working off of the correct set-up, it worked for most of the questions, but I can't seem to get number 23 from here.

Thanks in advance!

Beth
 Emily Haney-Caron
PowerScore Staff
  • PowerScore Staff
  • Posts: 577
  • Joined: Jan 12, 2012
|
#28647
Hi Beth,

Great job on the setups and the templates!! Sounds like you have it correct. For 23, you're working off your templates, and trying to identify the answer choice that cannot occur in either template.

A: F can be first in either template, so rule this one out.

B: G can be 5th (behind M, H, F, and L) in the first template you identified, so rule this one out.

C: H can be third in either template (behind M and F or L in the first, and behind L and F in the second), so this is out.

D: L can be second in either template (behind either F or M in the first, and behind F in the second), so this is out.

E: In the first template, M can't be 4th, because H, G, and K have to come after it. In the second template, every other variable has to come before M. So, this is the answer!
User avatar
 andy12
  • Posts: 9
  • Joined: Jun 22, 2023
|
#105604
Hi Powerscore team,

I did this game without templates without issue until this question. I definitely see the value in templates, but I didn't go for them right away. I'm wondering what you think about how to approach this question in this situation (not having already written down two templates). By looking at previous work I easily eliminated B and C (q. 18 proves both wrong because they're possible), but then drew three hypotheticals to verify E was impossible. Short of stopping at two and trusting eliminating two answers (B and C) via elimination and two (A and D) via hypotheticals, is there a faster way to look at this question without templates?

Thanks!
User avatar
 Dana D
PowerScore Staff
  • PowerScore Staff
  • Posts: 68
  • Joined: Feb 06, 2024
|
#105620
Hey Andy,

The benefit of templates is that they are the faster way of looking at the hypotheticals - no matter what questions you get, there are only two ways this game can work - with M :longline: H, K or with K :longline: M. Taking the time to diagram this and make a template lets you see inferences you might not otherwise and will let you work more quickly through questions such as this one.
User avatar
 andy12
  • Posts: 9
  • Joined: Jun 22, 2023
|
#105627
thanks, Dana!

Get the most out of your LSAT Prep Plus subscription.

Analyze and track your performance with our Testing and Analytics Package.